LSAT and Law School Admissions Forum

Get expert LSAT preparation and law school admissions advice from PowerScore Test Preparation.

 Administrator
PowerScore Staff
  • PowerScore Staff
  • Posts: 8917
  • Joined: Feb 02, 2011
|
#59632
Complete Question Explanation
(The complete setup for this game can be found here: lsat/viewtopic.php?t=1624)

The correct answer choice is (A)

This question also trades on the Numerical Distribution established by the rules, so let’s take a moment to revisit that Numerical Distribution:

  • F: F serves exactly two buildings.
    H: H could serve one, two, or three buildings.
    I: I must serve at least two buildings, and possibly all three buildings.
    P: P can only serve one building.
    S: S is limited to serving either one or two buildings.
    T: T serves exactly two buildings.

The distribution proves that only H and I can serve all three buildings, and thus answer choice (A) is correct.

Answer choice (A): This is the correct answer choice.

Answer choice (B): Because S can serve only one or two buildings, this answer choice is incorrect.

Answer choice (C): Because T serves exactly two buildings, this answer choice is incorrect.

Answer choice (D): Because P serves exactly one building, this answer choice is incorrect.

Answer choice (E): Because P serves exactly one building, and S can serve only one or two buildings, this answer choice is incorrect.

Get the most out of your LSAT Prep Plus subscription.

Analyze and track your performance with our Testing and Analytics Package.